LSAT and Law School Admissions Forum

Get expert LSAT preparation and law school admissions advice from PowerScore Test Preparation.

User avatar
 Dave Killoran
PowerScore Staff
  • PowerScore Staff
  • Posts: 5849
  • Joined: Mar 25, 2011
|
#86837
Complete Question Explanation
(The complete setup for this game can be found here: lsat/viewtopic.php?f=137&t=12861)

The correct answer choice is (D)

If J’s session is 3rd, then L’s session must be 1st, and from the third rule we can infer that M’s session is 2nd. H, and F :longline: GK remain to be placed. H cannot 4th due to the first rule, and so F must be on the 4th:


PT61_O2010 LG Explanations-g4_q20_d1.png

As G’s session can be on the 5th, answer choice (D) could be true and is correct.
 ekraft
  • Posts: 2
  • Joined: Aug 16, 2013
|
#10309
I don't understand how D could be true, if rule number 1 requires H/M__ __ M/H. With the identified "correct" answer this rule would be broken by: M J F G K H. How could this be true?


Question #20: Local, Could Be True. The correct answer choice is (D)
If J’s session is 3rd, then L’s session must be 1st, and from the third rule we can infer that M’s session is 2nd. H, and F > GK remain to be placed. H cannot 4th due to the first rule, and so F must be on the 4th:
As G’s session can be on the 5th, answer choice (D) could be true and is correct.

Kind regards,
Elaine
 Ron Gore
PowerScore Staff
  • PowerScore Staff
  • Posts: 220
  • Joined: May 15, 2013
|
#10310
Thank you for your question, Elaine.

Regarding Q.20, it appears you have misread the rule regarding H and M. The rule stated that "at least two of the other nurses’ sessions must fall in between Heany’s session and Moreau’s session." Your question indicates you perceived the rule to mean there are exactly two other sessions in between. Please let me know if I have misunderstood your question.

Thanks,

Ron
 ekraft
  • Posts: 2
  • Joined: Aug 16, 2013
|
#10323
Thanks Ron

I had missed exactly as pointed out "at least". I need some more practice :-D

Kind regards,
Elaine
 pineron11@gmail.com
  • Posts: 1
  • Joined: Nov 29, 2021
|
#92361
Hello,

I am trying to understand why L must be in the first session? Was this determined from an inference or a rule?
 Adam Tyson
PowerScore Staff
  • PowerScore Staff
  • Posts: 5153
  • Joined: Apr 14, 2011
|
#92366
It is indeed from an inference, pineron11@gmail.com! L can only ever be either 1st or 3rd, and if J is 3rd in this local scenario then L must be 1st. The setup that includes that inference about L can be found at this thread:

viewtopic.php?f=137&t=12861

L cannot be in spots 5, 6, or 7 because it must be before F and the GK block. It cannot be 2nd because the last rule says so. So why not 4th? Because if you place it 4th, then the last 4 spaces would be filled with LFGK, and that would only leave the first 3 spaces for MJH, but M and H must have at least two spaces between them per the first rule! It's also the reason that the GK block cannot be in spots 3 and 4 - that would mean LFGK fill the first 4 spaces, squeezing M and H to close to each other at the end of the sequence.
User avatar
 christinecwt
  • Posts: 74
  • Joined: May 09, 2022
|
#97839
Hi Team - May I know why Answer Choice C is wrong? Many thanks! Isn't that G can be 5th, H can be 6th, while K can be the last one? Thanks!

Get the most out of your LSAT Prep Plus subscription.

Analyze and track your performance with our Testing and Analytics Package.